Answer

问题及解答

设 $f(x)$ 是 $[0,1]$ 上的二次连续可微函数, 满足 $f(0)=f(1)=0$, $f'(0)=1$, $f'(1)=0$. 证明 $\int_0^1 (f''(x))^2 dx\geqslant 4$.

Posted by haifeng on 2017-06-23 18:08:42 last update 2022-04-19 09:41:03 | Edit | Answers (1)

设 $f(x)$ 是 $[0,1]$ 上的二次连续可微函数, 满足 $f(0)=f(1)=0$, $f'(0)=1$, $f'(1)=0$. 证明

\[
\int_0^1 (f''(x))^2 dx\geqslant 4.
\]

 

 


相关问题:

问题1473

问题1911  问题1912  问题1913  问题1863

1

Posted by haifeng on 2017-06-23 18:55:29

设 $0 < x_0\leqslant 1$. 注意到

\[
\Bigl((x-x_0)f'(x)-f(x)\Bigr)'=f'(x)+(x-x_0)f''(x)-f'(x)=(x-x_0)f''(x),
\]

因此,

\[
\begin{split}
\int_0^1 (x-x_0)f''(x)dx&=\Bigl[(x-x_0)f'(x)-f(x)\Bigr]\biggr|_0^1\\
&=\bigl[(1-x_0)f'(1)-f(1)\bigr]-\bigl[(0-x_0)f'(0)-f(0)\bigr]\\
&=x_0.
\end{split}
\]

因此, 由 Hölder 不等式

\[
\begin{split}
x_0^2=\biggl(\int_0^1 (x-x_0)f''(x)dx\biggr)^2 &\leqslant\int_0^1 (x-x_0)^2 dx\cdot\int_0^1(f''(x))^2 dx\\
&=\frac{1}{3}(x-x_0)^3\biggl|_0^1 \cdot\int_0^1(f''(x))^2 dx\\
&=\frac{(1-x_0)^3+x_0^3}{3}\cdot\int_0^1(f''(x))^2 dx\\
\end{split}
\]

这推出

\[
\begin{split}
\int_0^1(f''(x))^2 dx&\geqslant\frac{3x_0^2}{(1-x_0)^3+x_0^3}\\
&=\frac{3x_0^2}{(1-3x_0+3x_0^2-x_0^3)+x_0^3}\\
&=\frac{3}{\frac{1}{x_0^2}-\frac{3}{x_0}+3}\\
&=\frac{3}{(\frac{1}{x_0}-\frac{3}{2})^2+\frac{3}{4}}\\
\end{split}
\]

注意到 $x_0$ 是某个取定的数, 特别的, 我们取 $x_0=\frac{2}{3}$(可以在上述过程中将 $x_0$ 统一替换为 $\frac{2}{3}$), 则可得

\[
\int_0^1(f''(x))^2 dx\geqslant\frac{3}{\frac{3}{4}}=4.
\]


Remark:

This proof is provided by Shouwen FANG (方守文).